Arthur Mattuck, Introduction to Analysis, Problem 5-7

Click For Summary
The discussion revolves around proving that the recursively defined sequence a_{n+1} = √(2a_n) with a_0 > 0 is monotone and bounded. It is established that the sequence is decreasing for a_0 > 2 and increasing for 0 < a_0 < 2. The ratio a_{n+1}/a_n leads to the expression 2^{(1/2)^{n+1}}√a_0, which helps in determining the monotonicity based on the value of a_0. The key inequality to prove is that √(2x) > x, which helps identify the conditions under which the sequence is decreasing. Ultimately, the sequence is shown to be bounded and converges to the limit of 2.
EMR
Messages
3
Reaction score
0
This question is from Arthur Mattuck's "Introduction to Analysis", chapter 5, problem 5-7.

Homework Statement


Define a sequence recursively by a_{n+1}=\sqrt{2a_{n}}, a_{0}&gt;0.

(a) Prove that for any choice of a_{0}&gt;0, the sequence is monotone and bounded.


Homework Equations



None

The Attempt at a Solution



I've rewritten the sequence as

a_n = \sqrt{2{\sqrt{2\sqrt{2\sqrt{2...\sqrt{2a_0}}}}},

and taken the ratio of a_{n+1} and a_{n}, which leads me to the expression

\frac{a_{n+1}}{a_n} = 2^{(1/2)^{n+1}}\sqrt{a_0}.

From computing the sequence for a few initial values of a_0, I've been able to determine that the sequence is decreasing if a_0&gt;2 and increasing if 0&lt;a_0&lt;2. However, I'm not sure how to show this from this ratio, i.e. how does

2^{(1/2)^{n+1}}\sqrt{a_0}&lt;1

which indicates that the sequence is decreasing, turn into a_0&gt;2?

The rest of the problem comes easy enough. I'm able to show that it is bounded and that the limit is 2 (in part b of the question).
 
Physics news on Phys.org
You are making this too complicated. Proving a_(n+1)>a_n translates into sqrt(2*a_n)>a_n. For what range of x is it true that sqrt(2x)>x.
 
Question: A clock's minute hand has length 4 and its hour hand has length 3. What is the distance between the tips at the moment when it is increasing most rapidly?(Putnam Exam Question) Answer: Making assumption that both the hands moves at constant angular velocities, the answer is ## \sqrt{7} .## But don't you think this assumption is somewhat doubtful and wrong?

Similar threads

  • · Replies 4 ·
Replies
4
Views
2K
Replies
8
Views
3K
  • · Replies 6 ·
Replies
6
Views
2K
  • · Replies 12 ·
Replies
12
Views
2K
  • · Replies 9 ·
Replies
9
Views
2K
Replies
11
Views
2K
  • · Replies 1 ·
Replies
1
Views
2K
Replies
14
Views
2K
  • · Replies 13 ·
Replies
13
Views
2K
  • · Replies 1 ·
Replies
1
Views
1K